next up previous contents
Nächste Seite: Stetige Funktionen auf kompakten Aufwärts: Supremum und Zwischenwertsatz Vorherige Seite: Zwischenwertsatz   Inhalt

Stetigkeit der Umkehrfunktion

Satz 2.5.17 (Stetigkeit der Umkehrfunktion)  

Es seien $ I $, $ J \subset \mathbb{R}$ nichtausgeartete Intervalle und $ f:I\rightarrow J $ eine streng monoton wachsende Funktion mit Umkehrfunktion $ g: J\rightarrow I $.

Dann sind $ f$ und $ g $ stetig.

Anmerkung

  1. Der Satz gilt analog für streng monoton fallende Funktionen.
  2. Wie die folgenden beiden Beispiele zeigen, benötigt man beide Voraussetungen:

Beispiele

  1. Man setze $ g = f : [-1,1) \rightarrow [-1,1) $ mit:

    $\displaystyle g(x) = f(x) = \left\{\begin{array}{ll}
x+1 &\text{wenn \( -1 \leq...
... x < 0 \),}\\
x-1 &\text{wenn \( 0 \leqslant x < 1 \).}
\end{array} \right.
$

    Es ist $ g = f^{-1}$ aber unstetig in $ x_0=0 $.
  2. Man definiere $ f: [-1,1] \rightarrow [-1,0)\cup[1,2] $ mit Umkehrfunktion $ g:[-1,0)\cup[1,2] \rightarrow [-1,1] $ durch

    $\displaystyle f(x)$ $\displaystyle :=\left\{ \begin{array}{ll} x &\text{wenn \( -1 \leqslant x < 0 \),}\\ x+1 &\text{wenn \( 0\leqslant x \leqslant 1 \).} \end{array} \right.$    
    $\displaystyle g(y)$ $\displaystyle :=\left\{ \begin{array}{ll} y &\text{wenn \( -1 \leqslant y < 0 \),}\\ y-1 &\text{wenn \( 1 \leqslant y \leqslant 2 \).} \end{array} \right.$    

    Die streng monotone Funktion $ f$ hat eine Sprungstelle in $ x_0=0 $.

Beweis (Stetigkeit der Umkehrfunktion).

Wir zeigen die Stetigkeit der Umkehrfunktion $ g $ (vgl. Satz [*]).

Es seien $ x_0\in I $, $ y_0=f(x_0)\in J $ und $ \varepsilon >0$. Es sei $ a\in \overline{R} $ der linke Endpunkt von $ I $ und $ b \in \overline{\mathbb{R}} $ der rechte Endpunkt. Man setze

$\displaystyle \alpha$ $\displaystyle :=\left\{ \begin{array}{ll} x_0-\varepsilon & \text{falls \( x_0-\varepsilon \in I \),}\\ \frac{1}{2}(a+x_0) & \text{sonst;} \end{array} \right.$    
$\displaystyle <tex2html_comment_mark>279 \beta$ $\displaystyle :=\left\{ \begin{array}{ll} x_0+\varepsilon & \text{falls \( x_0+\varepsilon \in I \),}\\ \frac{1}{2}(x_0+b) & \text{sonst;} \end{array} \right.$    
$\displaystyle <tex2html_comment_mark>280 \delta$ $\displaystyle :=\left\{ \begin{array}{ll} \min\{ y_0-f(\alpha), f(\beta)-y_0 \}...
...= x_0 \),}\\ y_0-\alpha &\text{wenn \( x_0 = \beta = b \).} \end{array} \right.$    

Es ist $ \delta>0$. Für $ y\in J $ gilt

$\displaystyle \vert y-y_0 \vert\leqslant \delta
\quad\Rightarrow\quad f(\alpha...
...qslant f(\beta)
\quad\Leftrightarrow\quad \alpha \leqslant g(y) \leqslant \beta$   .$\displaystyle $

und somit $ \vert g(y_0)-g(y)\vert = \vert x_0-g(y)\vert \leqslant \varepsilon $.

Aus Satz [*] und Korollar [*] folgt nun:

Korollar 2.5.18 (Stetigkeit der Umkehrfuntion)  

Sei $ I\subset \mathbb{R}$ ein Intervall und $ f:I \rightarrow \mathbb{R}$ eine stetige und streng monotone Funktion. Dann ist die Umkehrfunktion $ f^{-1}:J\rightarrow\mathbb{R}$ stetig, wobei $ J:=f(I)$.

Satz 2.5.19   Sei $ I\subset \mathbb{R}$ ein nicht ausgeartetes Intervall und $ f:I \rightarrow \mathbb{R}$ eine stetige und injektive Funktion. Dann ist $ f$ streng monoton.

Wir betrachten zunächst den Fall $ I = [a,b] $.

Lemma 2.5.20   Es seien $ a$, $ b \in \mathbb{R}$, $ a < b$ und $ f: [a,b] \rightarrow \mathbb{R}$ injektiv und stetig.

Wenn $ f(a) < f(b) $, so ist $ f$ streng monoton wachsend.

Beweis . Annahme: $ f$ ist nicht strikt monoton wachsend:

Dann gibt es Punkte $ x$, $ y\in [a,b] $ mit $ x<y$ und $ f(x)\geqslant f(y) $. Da $ f$ injektiv, ist $ f(x) > f(y)$. Wir unterscheiden zwei Fälle:

\fbox{\( f(a)\leqslant f(y) \):}
Zu einem $ c$ mit $ f(y) < c < f(x) $ gibt es nach dem Zwischenwertsatz [*]

$\displaystyle \xi \in (a,x)$   und$\displaystyle \quad \eta \in (x,y)
$

mit $ f(\xi)=f(c) = f(\eta) $.

Da $ \xi\not=\eta $, widerspricht das der Injektivität von $ f$.

\fbox{\( f(y) < f(a) \):}
Zu einem $ c$ mit $ f(y) < c < f(a) $ gibt es nach dem Zwischenwertsatz [*]

$\displaystyle \xi \in (a,y))$   und$\displaystyle \quad \eta \in (y,b)
$

mit $ f(\xi)=c=f(\eta) $.

Da $ \xi\not=\eta $, widerspricht das der Injektivität von $ f$.

Beweis . Wähle $ a$, $ b \in I $ mit $ a < b$.

Wir zeigen, wenn $ f(a) < f(b) $, dann ist $ f$ streng monoton wachsend:

Seien $ x$, $ y \in I $ mit $ x<y$. Man setze

$\displaystyle c:=\min \{ x,a \}$   und$\displaystyle \quad d :=\max \{ y,b \}$.$\displaystyle $

Nach dem vorangehenden Lemma ist die Einschänkung $ f\vert _{[c,d]} $ entweder streng monoton wachsend oder fallend, je nachdem ob $ f(c) < f(d) $ oder $ f(c)>f(d) $ ist.

Da $ a,b \in [c,d] $ und $ f(a) < f(b) $ ist, muß die Einschränkung $ f\vert _{[c,d]} $ streng monoton wachsend sein.

Da $ x,y \in [c,d] $ ist, folgt $ f(x) < f(y)$.
Wenn $ f(a) > f(b) $ ist, folgt analog, daß $ f$ streng monoton fallend ist.

Beispiel (Logarithmus).

Für $ a \in \mathbb{R}$, $ a>1 $ ist die Exponentialfunktion zur Basis $ a$:

$\displaystyle \mathbb{R}\ni x \mapsto a^x \in (0,\infty)
$

stetig, streng monoton wachsend und bijektiv.

Für $ a \in \mathbb{R}$, $ 0 < a < 1 $ ist die Exponentialfunktion zur Basis $ a$:

$\displaystyle \mathbb{R}\ni x \mapsto a^x \in (0,\infty)
$

stetig, streng monoton fallend und bijektiv.

Definition 2.5.21 (Logarithmus zur Basis $ a$)  

Es sei $ a \in \mathbb{R}$, $ 0<a $. Die Umkehrfunktion der Exponentialfunktion zur Basis $ a$ heißt Logarithmus zur Basis $ a$:

$\displaystyle (0,\infty) \ni x \mapsto \log_a(x) \in \mathbb{R}$.$\displaystyle $

Bezeichnung 2.5.22 (Der natürliche Logarithmus)  

Die Umkehrfunktion der Exponentialfunktion $ \exp: x \mapsto e^x $ heißt natürlicher Logarithmus oder kurz der Logarithmus und wird mit

$\displaystyle \log x$   oder$\displaystyle \quad \ln x
$

bezeichnet.

Mathematiker schreiben meistens $ \log x $, Physiker und Ingenieure $ \ln x $.

Bemerkung 2.5.23  
  1. Für $ a \in \mathbb{R}$, $ a>0 $ gilt

    $\displaystyle a^x = e^{x\,\log a}$   für $ x \in \mathbb{R}$.$\displaystyle $

  2. Die Logarithmen zu verschiedenen Basen unterscheiden sich nur um einen konstanten Faktor:

    $\displaystyle \log_a x = \frac{\log x}{\log a}$   für $ 0<a $, $ a\not=1 $ .$\displaystyle $

    Daher reicht es, den natürlichen Logarithmus zu untersuchen.

Feststellung 2.5.24 (Regeln: Logarithmus)  

  1. Der Logarithmus $ \log: (0,\infty) \rightarrow \mathbb{R}$ ist stetig, streng monoton wachsend und streng konkav.
  2. Es gilt die Funktionalgleichung

    $\displaystyle \log(xy) = \log x + \log y$   für $ x$, $ y\in (0,\infty) $.$\displaystyle $

  3. Es gilt

    $\displaystyle \lim_{x\downarrow 0}\log x = -\infty, \quad
\log 1 = 0,~
\log e = 1,\quad
\lim_{x\to\infty} \log x = \infty$   .$\displaystyle $

Anmerkung. Wir werden noch weitere wichtige Eigenschaften der Exponentialfunktion und des Logarithmus in den folgenden Kapiteln kennenlernen.


next up previous contents
Nächste Seite: Stetige Funktionen auf kompakten Aufwärts: Supremum und Zwischenwertsatz Vorherige Seite: Zwischenwertsatz   Inhalt
Analysis1-A.Lambert 2001-02-09